A continuous function on $[0,1]$ that is not of a bounded veriation











up vote
4
down vote

favorite
1












Suppose $f$ is continuous on [0, 1]. Must there be a nondegenerate closed subinterval $[a, b]$
of [0, 1] for which the restriction of $f$ to $[a, b]$ is of bounded variation?



$mathbf{My attempt}:$
Suppose for all $[a,b]subseteq [0,1]$ with $aneq b$, we have $f$ restricted to $[a,b]$ is not of a bounded variation, $i.e$ for all $n in mathbb{Z^+}$ there exists a partition $mathcal{P}_{{n}}$ such that $V(f,mathcal{P_n})geq n$.



I am not sure if there would be some examples shows that is not true, I am using a contradiction here to conclude that there must be a nondegenerate interval of $[0,1$ such that $f$ restricted to this interval is of a bounded variation. I will appreciate it any help with that. Thank you.










share|cite|improve this question
























  • So to be clear, you are asking if there is a continuous function on the unit interval that is not of bounded variation on any sub-interval?
    – Valborg
    Nov 25 at 22:27












  • Yes, I am trying to show that there must be at least one non degenerated interval such that $f$ restricted to this interval is of a bounded variation because so far as I think there should exists by continuity. However, I am not sure if it can be done by showing example for a continuous function on $[0,1]$ such that $f$ restricted to any nondegenerate interval is not of a bounded variation.
    – Ahmed
    Nov 25 at 22:36










  • Sorry my attempt was confusing, I modify it a little bit.
    – Ahmed
    Nov 25 at 22:39















up vote
4
down vote

favorite
1












Suppose $f$ is continuous on [0, 1]. Must there be a nondegenerate closed subinterval $[a, b]$
of [0, 1] for which the restriction of $f$ to $[a, b]$ is of bounded variation?



$mathbf{My attempt}:$
Suppose for all $[a,b]subseteq [0,1]$ with $aneq b$, we have $f$ restricted to $[a,b]$ is not of a bounded variation, $i.e$ for all $n in mathbb{Z^+}$ there exists a partition $mathcal{P}_{{n}}$ such that $V(f,mathcal{P_n})geq n$.



I am not sure if there would be some examples shows that is not true, I am using a contradiction here to conclude that there must be a nondegenerate interval of $[0,1$ such that $f$ restricted to this interval is of a bounded variation. I will appreciate it any help with that. Thank you.










share|cite|improve this question
























  • So to be clear, you are asking if there is a continuous function on the unit interval that is not of bounded variation on any sub-interval?
    – Valborg
    Nov 25 at 22:27












  • Yes, I am trying to show that there must be at least one non degenerated interval such that $f$ restricted to this interval is of a bounded variation because so far as I think there should exists by continuity. However, I am not sure if it can be done by showing example for a continuous function on $[0,1]$ such that $f$ restricted to any nondegenerate interval is not of a bounded variation.
    – Ahmed
    Nov 25 at 22:36










  • Sorry my attempt was confusing, I modify it a little bit.
    – Ahmed
    Nov 25 at 22:39













up vote
4
down vote

favorite
1









up vote
4
down vote

favorite
1






1





Suppose $f$ is continuous on [0, 1]. Must there be a nondegenerate closed subinterval $[a, b]$
of [0, 1] for which the restriction of $f$ to $[a, b]$ is of bounded variation?



$mathbf{My attempt}:$
Suppose for all $[a,b]subseteq [0,1]$ with $aneq b$, we have $f$ restricted to $[a,b]$ is not of a bounded variation, $i.e$ for all $n in mathbb{Z^+}$ there exists a partition $mathcal{P}_{{n}}$ such that $V(f,mathcal{P_n})geq n$.



I am not sure if there would be some examples shows that is not true, I am using a contradiction here to conclude that there must be a nondegenerate interval of $[0,1$ such that $f$ restricted to this interval is of a bounded variation. I will appreciate it any help with that. Thank you.










share|cite|improve this question















Suppose $f$ is continuous on [0, 1]. Must there be a nondegenerate closed subinterval $[a, b]$
of [0, 1] for which the restriction of $f$ to $[a, b]$ is of bounded variation?



$mathbf{My attempt}:$
Suppose for all $[a,b]subseteq [0,1]$ with $aneq b$, we have $f$ restricted to $[a,b]$ is not of a bounded variation, $i.e$ for all $n in mathbb{Z^+}$ there exists a partition $mathcal{P}_{{n}}$ such that $V(f,mathcal{P_n})geq n$.



I am not sure if there would be some examples shows that is not true, I am using a contradiction here to conclude that there must be a nondegenerate interval of $[0,1$ such that $f$ restricted to this interval is of a bounded variation. I will appreciate it any help with that. Thank you.







measure-theory lebesgue-measure






share|cite|improve this question















share|cite|improve this question













share|cite|improve this question




share|cite|improve this question








edited Nov 25 at 22:38

























asked Nov 25 at 22:24









Ahmed

29019




29019












  • So to be clear, you are asking if there is a continuous function on the unit interval that is not of bounded variation on any sub-interval?
    – Valborg
    Nov 25 at 22:27












  • Yes, I am trying to show that there must be at least one non degenerated interval such that $f$ restricted to this interval is of a bounded variation because so far as I think there should exists by continuity. However, I am not sure if it can be done by showing example for a continuous function on $[0,1]$ such that $f$ restricted to any nondegenerate interval is not of a bounded variation.
    – Ahmed
    Nov 25 at 22:36










  • Sorry my attempt was confusing, I modify it a little bit.
    – Ahmed
    Nov 25 at 22:39


















  • So to be clear, you are asking if there is a continuous function on the unit interval that is not of bounded variation on any sub-interval?
    – Valborg
    Nov 25 at 22:27












  • Yes, I am trying to show that there must be at least one non degenerated interval such that $f$ restricted to this interval is of a bounded variation because so far as I think there should exists by continuity. However, I am not sure if it can be done by showing example for a continuous function on $[0,1]$ such that $f$ restricted to any nondegenerate interval is not of a bounded variation.
    – Ahmed
    Nov 25 at 22:36










  • Sorry my attempt was confusing, I modify it a little bit.
    – Ahmed
    Nov 25 at 22:39
















So to be clear, you are asking if there is a continuous function on the unit interval that is not of bounded variation on any sub-interval?
– Valborg
Nov 25 at 22:27






So to be clear, you are asking if there is a continuous function on the unit interval that is not of bounded variation on any sub-interval?
– Valborg
Nov 25 at 22:27














Yes, I am trying to show that there must be at least one non degenerated interval such that $f$ restricted to this interval is of a bounded variation because so far as I think there should exists by continuity. However, I am not sure if it can be done by showing example for a continuous function on $[0,1]$ such that $f$ restricted to any nondegenerate interval is not of a bounded variation.
– Ahmed
Nov 25 at 22:36




Yes, I am trying to show that there must be at least one non degenerated interval such that $f$ restricted to this interval is of a bounded variation because so far as I think there should exists by continuity. However, I am not sure if it can be done by showing example for a continuous function on $[0,1]$ such that $f$ restricted to any nondegenerate interval is not of a bounded variation.
– Ahmed
Nov 25 at 22:36












Sorry my attempt was confusing, I modify it a little bit.
– Ahmed
Nov 25 at 22:39




Sorry my attempt was confusing, I modify it a little bit.
– Ahmed
Nov 25 at 22:39










2 Answers
2






active

oldest

votes

















up vote
3
down vote













It is well known that any function of bounded variation is differentiable almost everywhere and that there exists a function that is continuous but differentiable nowhere. For the first statement use the fact that any function of bounded variation is the difference of two monotone functions and monotone functions are differentiable almost everywhere: http://mathonline.wikidot.com/lebesgue-s-theorem-for-the-differentiability-of-monotone-fun



For the second statement go to https://en.wikipedia.org/wiki/Weierstrass_function






share|cite|improve this answer























  • It might be helpful for you to elaborate just a tad on the logical implications here, as well as include an example of such a function. I like en.wikipedia.org/wiki/Blancmange_curve.
    – Valborg
    Nov 25 at 23:56


















up vote
1
down vote













Hint: it is well known that functions of bounded variation are of the form $f = g-h$ with $g,h$ monotone. In particular, $f$ ought to be differentiable almost everywhere on $[0,1]$. However, functions such as the Weierstraß function (restricted to $[0,1]$) are continuous but differentiable nowhere. In particular it is not differentiable in any proper subinterval and so by the former, it is not of bounded variation there either.



Edit: the original question was modified, and this does not answer the content of the post: what follows is a construction of a continuous function $f : [0,1] to mathbb{R}$ which is not of bounded variation in any subinterval $[0,c]$ with $c leq 1$, but for which $V_c^1f < +infty$.



First, let's construct a function $[0,1] to mathbb{R}$ which is not of bounded variation.



Consider $f_N : left[frac{1}{N+1},frac{1}{N}right]to mathbb{R}$ with interpolates linearly the points:
$$
(1/(N+1),0),left(frac{1/(N+1)+1/N}{2},1/Nright) text{ and } (1/N,0)
$$



A drawing should illustrate the idea: we are making spikes whose height decreases with $N$, and so does the length of the considered interval. At the endpoint of the interval $f_N$ is zero, and in the middle point it attains its maximum value which is $1/N$. Moreover, $f_N$ is piecewise monotone. Now the functions $(f_N)_{Ngeq 1}$ coincide at ${1/N : N in mathbb{N}}$ and so the function



$$
f(x) = f_N(x) text{, if } x in left[frac{1}{N+1},frac{1}{N}right]
$$



is continuous on $(0,1]$. Technicalities aside, we are just gluing the spikes together and justifying that the resulting function is indeed contiunous. Finally, we would like to extend $f$ to $[0,1]$. A necessary condition is for $f(0)$ to be zero, since we already have $f(1/N) = 0$ and $1/N to 0$. So let's check that



$$
f(x) = cases{f(x) x neq 0 \ 0 text{ otherwise}}
$$



is continuous on $[0,1]$. Take $x_n to 0$ and $varepsilon > 0$. Since $x_n$ goes to zero, at some point $n_0$ we have that $|x_n| < varepsilon$ if $n geq n_0$. Now, if $n geq n_0$, each $x_n$ is either zero, lies on some interval $left[frac{1}{M_n+1},frac{1}{M_n}right]$ with $M_n > 1/varepsilon$, and so by the definition of each $f_N$ then, we have that $|f(x_n)| = f(x_n) leq 1/M_n < varepsilon$. In any case, we have that $|f(x_n)| < varepsilon$ if $n geq n_0$, which proves that $f$ is continuous at zero.



So far we have only defined $f$. We still ought to show that it is not of bounded variation, despite being continuous. Intuitively this is because the spikes add each a variation of no less that $1/N$ and the harmonic series diverges. Concretely, if $f$ were of bounded variation, we would have that for each $N geq 1$,



$$
V_0^1f = V_0^{1/(N+1)}f + sum_{i=1}^NV_{1/(i+1)}^{1/i}f = dots \
dots = V_0^{1/(N+1)}f + sum_{i=1}^NV_{1/(i+1)}^{1/i}f_i geq V_0^{1/(N+1)}f + sum_{i=1}^Nfrac{1}{i} geq sum_{i=1}^Nfrac{1}{i}.
$$



Taking limit when $i to infty$ we have $V_0^1f = +infty$. Moreover, $f$ is not of bounded variation in any interval of the form $[0,c]$: if it were so, by refining partitions and taking limits we would have that



$$
V_0^1f = V_0^cf + V_c^1f,
$$



and since $f|_{[c,1]}$ is of bounded variation (we are interpolating linearly a finite amount of times), then $V_c^1 < + infty$ which would imply that $V_0^1f < + infty$, a contradiction.






share|cite|improve this answer























    Your Answer





    StackExchange.ifUsing("editor", function () {
    return StackExchange.using("mathjaxEditing", function () {
    StackExchange.MarkdownEditor.creationCallbacks.add(function (editor, postfix) {
    StackExchange.mathjaxEditing.prepareWmdForMathJax(editor, postfix, [["$", "$"], ["\\(","\\)"]]);
    });
    });
    }, "mathjax-editing");

    StackExchange.ready(function() {
    var channelOptions = {
    tags: "".split(" "),
    id: "69"
    };
    initTagRenderer("".split(" "), "".split(" "), channelOptions);

    StackExchange.using("externalEditor", function() {
    // Have to fire editor after snippets, if snippets enabled
    if (StackExchange.settings.snippets.snippetsEnabled) {
    StackExchange.using("snippets", function() {
    createEditor();
    });
    }
    else {
    createEditor();
    }
    });

    function createEditor() {
    StackExchange.prepareEditor({
    heartbeatType: 'answer',
    convertImagesToLinks: true,
    noModals: true,
    showLowRepImageUploadWarning: true,
    reputationToPostImages: 10,
    bindNavPrevention: true,
    postfix: "",
    imageUploader: {
    brandingHtml: "Powered by u003ca class="icon-imgur-white" href="https://imgur.com/"u003eu003c/au003e",
    contentPolicyHtml: "User contributions licensed under u003ca href="https://creativecommons.org/licenses/by-sa/3.0/"u003ecc by-sa 3.0 with attribution requiredu003c/au003e u003ca href="https://stackoverflow.com/legal/content-policy"u003e(content policy)u003c/au003e",
    allowUrls: true
    },
    noCode: true, onDemand: true,
    discardSelector: ".discard-answer"
    ,immediatelyShowMarkdownHelp:true
    });


    }
    });














    draft saved

    draft discarded


















    StackExchange.ready(
    function () {
    StackExchange.openid.initPostLogin('.new-post-login', 'https%3a%2f%2fmath.stackexchange.com%2fquestions%2f3013501%2fa-continuous-function-on-0-1-that-is-not-of-a-bounded-veriation%23new-answer', 'question_page');
    }
    );

    Post as a guest















    Required, but never shown

























    2 Answers
    2






    active

    oldest

    votes








    2 Answers
    2






    active

    oldest

    votes









    active

    oldest

    votes






    active

    oldest

    votes








    up vote
    3
    down vote













    It is well known that any function of bounded variation is differentiable almost everywhere and that there exists a function that is continuous but differentiable nowhere. For the first statement use the fact that any function of bounded variation is the difference of two monotone functions and monotone functions are differentiable almost everywhere: http://mathonline.wikidot.com/lebesgue-s-theorem-for-the-differentiability-of-monotone-fun



    For the second statement go to https://en.wikipedia.org/wiki/Weierstrass_function






    share|cite|improve this answer























    • It might be helpful for you to elaborate just a tad on the logical implications here, as well as include an example of such a function. I like en.wikipedia.org/wiki/Blancmange_curve.
      – Valborg
      Nov 25 at 23:56















    up vote
    3
    down vote













    It is well known that any function of bounded variation is differentiable almost everywhere and that there exists a function that is continuous but differentiable nowhere. For the first statement use the fact that any function of bounded variation is the difference of two monotone functions and monotone functions are differentiable almost everywhere: http://mathonline.wikidot.com/lebesgue-s-theorem-for-the-differentiability-of-monotone-fun



    For the second statement go to https://en.wikipedia.org/wiki/Weierstrass_function






    share|cite|improve this answer























    • It might be helpful for you to elaborate just a tad on the logical implications here, as well as include an example of such a function. I like en.wikipedia.org/wiki/Blancmange_curve.
      – Valborg
      Nov 25 at 23:56













    up vote
    3
    down vote










    up vote
    3
    down vote









    It is well known that any function of bounded variation is differentiable almost everywhere and that there exists a function that is continuous but differentiable nowhere. For the first statement use the fact that any function of bounded variation is the difference of two monotone functions and monotone functions are differentiable almost everywhere: http://mathonline.wikidot.com/lebesgue-s-theorem-for-the-differentiability-of-monotone-fun



    For the second statement go to https://en.wikipedia.org/wiki/Weierstrass_function






    share|cite|improve this answer














    It is well known that any function of bounded variation is differentiable almost everywhere and that there exists a function that is continuous but differentiable nowhere. For the first statement use the fact that any function of bounded variation is the difference of two monotone functions and monotone functions are differentiable almost everywhere: http://mathonline.wikidot.com/lebesgue-s-theorem-for-the-differentiability-of-monotone-fun



    For the second statement go to https://en.wikipedia.org/wiki/Weierstrass_function







    share|cite|improve this answer














    share|cite|improve this answer



    share|cite|improve this answer








    edited Nov 26 at 0:01

























    answered Nov 25 at 23:20









    Kavi Rama Murthy

    46.3k31854




    46.3k31854












    • It might be helpful for you to elaborate just a tad on the logical implications here, as well as include an example of such a function. I like en.wikipedia.org/wiki/Blancmange_curve.
      – Valborg
      Nov 25 at 23:56


















    • It might be helpful for you to elaborate just a tad on the logical implications here, as well as include an example of such a function. I like en.wikipedia.org/wiki/Blancmange_curve.
      – Valborg
      Nov 25 at 23:56
















    It might be helpful for you to elaborate just a tad on the logical implications here, as well as include an example of such a function. I like en.wikipedia.org/wiki/Blancmange_curve.
    – Valborg
    Nov 25 at 23:56




    It might be helpful for you to elaborate just a tad on the logical implications here, as well as include an example of such a function. I like en.wikipedia.org/wiki/Blancmange_curve.
    – Valborg
    Nov 25 at 23:56










    up vote
    1
    down vote













    Hint: it is well known that functions of bounded variation are of the form $f = g-h$ with $g,h$ monotone. In particular, $f$ ought to be differentiable almost everywhere on $[0,1]$. However, functions such as the Weierstraß function (restricted to $[0,1]$) are continuous but differentiable nowhere. In particular it is not differentiable in any proper subinterval and so by the former, it is not of bounded variation there either.



    Edit: the original question was modified, and this does not answer the content of the post: what follows is a construction of a continuous function $f : [0,1] to mathbb{R}$ which is not of bounded variation in any subinterval $[0,c]$ with $c leq 1$, but for which $V_c^1f < +infty$.



    First, let's construct a function $[0,1] to mathbb{R}$ which is not of bounded variation.



    Consider $f_N : left[frac{1}{N+1},frac{1}{N}right]to mathbb{R}$ with interpolates linearly the points:
    $$
    (1/(N+1),0),left(frac{1/(N+1)+1/N}{2},1/Nright) text{ and } (1/N,0)
    $$



    A drawing should illustrate the idea: we are making spikes whose height decreases with $N$, and so does the length of the considered interval. At the endpoint of the interval $f_N$ is zero, and in the middle point it attains its maximum value which is $1/N$. Moreover, $f_N$ is piecewise monotone. Now the functions $(f_N)_{Ngeq 1}$ coincide at ${1/N : N in mathbb{N}}$ and so the function



    $$
    f(x) = f_N(x) text{, if } x in left[frac{1}{N+1},frac{1}{N}right]
    $$



    is continuous on $(0,1]$. Technicalities aside, we are just gluing the spikes together and justifying that the resulting function is indeed contiunous. Finally, we would like to extend $f$ to $[0,1]$. A necessary condition is for $f(0)$ to be zero, since we already have $f(1/N) = 0$ and $1/N to 0$. So let's check that



    $$
    f(x) = cases{f(x) x neq 0 \ 0 text{ otherwise}}
    $$



    is continuous on $[0,1]$. Take $x_n to 0$ and $varepsilon > 0$. Since $x_n$ goes to zero, at some point $n_0$ we have that $|x_n| < varepsilon$ if $n geq n_0$. Now, if $n geq n_0$, each $x_n$ is either zero, lies on some interval $left[frac{1}{M_n+1},frac{1}{M_n}right]$ with $M_n > 1/varepsilon$, and so by the definition of each $f_N$ then, we have that $|f(x_n)| = f(x_n) leq 1/M_n < varepsilon$. In any case, we have that $|f(x_n)| < varepsilon$ if $n geq n_0$, which proves that $f$ is continuous at zero.



    So far we have only defined $f$. We still ought to show that it is not of bounded variation, despite being continuous. Intuitively this is because the spikes add each a variation of no less that $1/N$ and the harmonic series diverges. Concretely, if $f$ were of bounded variation, we would have that for each $N geq 1$,



    $$
    V_0^1f = V_0^{1/(N+1)}f + sum_{i=1}^NV_{1/(i+1)}^{1/i}f = dots \
    dots = V_0^{1/(N+1)}f + sum_{i=1}^NV_{1/(i+1)}^{1/i}f_i geq V_0^{1/(N+1)}f + sum_{i=1}^Nfrac{1}{i} geq sum_{i=1}^Nfrac{1}{i}.
    $$



    Taking limit when $i to infty$ we have $V_0^1f = +infty$. Moreover, $f$ is not of bounded variation in any interval of the form $[0,c]$: if it were so, by refining partitions and taking limits we would have that



    $$
    V_0^1f = V_0^cf + V_c^1f,
    $$



    and since $f|_{[c,1]}$ is of bounded variation (we are interpolating linearly a finite amount of times), then $V_c^1 < + infty$ which would imply that $V_0^1f < + infty$, a contradiction.






    share|cite|improve this answer



























      up vote
      1
      down vote













      Hint: it is well known that functions of bounded variation are of the form $f = g-h$ with $g,h$ monotone. In particular, $f$ ought to be differentiable almost everywhere on $[0,1]$. However, functions such as the Weierstraß function (restricted to $[0,1]$) are continuous but differentiable nowhere. In particular it is not differentiable in any proper subinterval and so by the former, it is not of bounded variation there either.



      Edit: the original question was modified, and this does not answer the content of the post: what follows is a construction of a continuous function $f : [0,1] to mathbb{R}$ which is not of bounded variation in any subinterval $[0,c]$ with $c leq 1$, but for which $V_c^1f < +infty$.



      First, let's construct a function $[0,1] to mathbb{R}$ which is not of bounded variation.



      Consider $f_N : left[frac{1}{N+1},frac{1}{N}right]to mathbb{R}$ with interpolates linearly the points:
      $$
      (1/(N+1),0),left(frac{1/(N+1)+1/N}{2},1/Nright) text{ and } (1/N,0)
      $$



      A drawing should illustrate the idea: we are making spikes whose height decreases with $N$, and so does the length of the considered interval. At the endpoint of the interval $f_N$ is zero, and in the middle point it attains its maximum value which is $1/N$. Moreover, $f_N$ is piecewise monotone. Now the functions $(f_N)_{Ngeq 1}$ coincide at ${1/N : N in mathbb{N}}$ and so the function



      $$
      f(x) = f_N(x) text{, if } x in left[frac{1}{N+1},frac{1}{N}right]
      $$



      is continuous on $(0,1]$. Technicalities aside, we are just gluing the spikes together and justifying that the resulting function is indeed contiunous. Finally, we would like to extend $f$ to $[0,1]$. A necessary condition is for $f(0)$ to be zero, since we already have $f(1/N) = 0$ and $1/N to 0$. So let's check that



      $$
      f(x) = cases{f(x) x neq 0 \ 0 text{ otherwise}}
      $$



      is continuous on $[0,1]$. Take $x_n to 0$ and $varepsilon > 0$. Since $x_n$ goes to zero, at some point $n_0$ we have that $|x_n| < varepsilon$ if $n geq n_0$. Now, if $n geq n_0$, each $x_n$ is either zero, lies on some interval $left[frac{1}{M_n+1},frac{1}{M_n}right]$ with $M_n > 1/varepsilon$, and so by the definition of each $f_N$ then, we have that $|f(x_n)| = f(x_n) leq 1/M_n < varepsilon$. In any case, we have that $|f(x_n)| < varepsilon$ if $n geq n_0$, which proves that $f$ is continuous at zero.



      So far we have only defined $f$. We still ought to show that it is not of bounded variation, despite being continuous. Intuitively this is because the spikes add each a variation of no less that $1/N$ and the harmonic series diverges. Concretely, if $f$ were of bounded variation, we would have that for each $N geq 1$,



      $$
      V_0^1f = V_0^{1/(N+1)}f + sum_{i=1}^NV_{1/(i+1)}^{1/i}f = dots \
      dots = V_0^{1/(N+1)}f + sum_{i=1}^NV_{1/(i+1)}^{1/i}f_i geq V_0^{1/(N+1)}f + sum_{i=1}^Nfrac{1}{i} geq sum_{i=1}^Nfrac{1}{i}.
      $$



      Taking limit when $i to infty$ we have $V_0^1f = +infty$. Moreover, $f$ is not of bounded variation in any interval of the form $[0,c]$: if it were so, by refining partitions and taking limits we would have that



      $$
      V_0^1f = V_0^cf + V_c^1f,
      $$



      and since $f|_{[c,1]}$ is of bounded variation (we are interpolating linearly a finite amount of times), then $V_c^1 < + infty$ which would imply that $V_0^1f < + infty$, a contradiction.






      share|cite|improve this answer

























        up vote
        1
        down vote










        up vote
        1
        down vote









        Hint: it is well known that functions of bounded variation are of the form $f = g-h$ with $g,h$ monotone. In particular, $f$ ought to be differentiable almost everywhere on $[0,1]$. However, functions such as the Weierstraß function (restricted to $[0,1]$) are continuous but differentiable nowhere. In particular it is not differentiable in any proper subinterval and so by the former, it is not of bounded variation there either.



        Edit: the original question was modified, and this does not answer the content of the post: what follows is a construction of a continuous function $f : [0,1] to mathbb{R}$ which is not of bounded variation in any subinterval $[0,c]$ with $c leq 1$, but for which $V_c^1f < +infty$.



        First, let's construct a function $[0,1] to mathbb{R}$ which is not of bounded variation.



        Consider $f_N : left[frac{1}{N+1},frac{1}{N}right]to mathbb{R}$ with interpolates linearly the points:
        $$
        (1/(N+1),0),left(frac{1/(N+1)+1/N}{2},1/Nright) text{ and } (1/N,0)
        $$



        A drawing should illustrate the idea: we are making spikes whose height decreases with $N$, and so does the length of the considered interval. At the endpoint of the interval $f_N$ is zero, and in the middle point it attains its maximum value which is $1/N$. Moreover, $f_N$ is piecewise monotone. Now the functions $(f_N)_{Ngeq 1}$ coincide at ${1/N : N in mathbb{N}}$ and so the function



        $$
        f(x) = f_N(x) text{, if } x in left[frac{1}{N+1},frac{1}{N}right]
        $$



        is continuous on $(0,1]$. Technicalities aside, we are just gluing the spikes together and justifying that the resulting function is indeed contiunous. Finally, we would like to extend $f$ to $[0,1]$. A necessary condition is for $f(0)$ to be zero, since we already have $f(1/N) = 0$ and $1/N to 0$. So let's check that



        $$
        f(x) = cases{f(x) x neq 0 \ 0 text{ otherwise}}
        $$



        is continuous on $[0,1]$. Take $x_n to 0$ and $varepsilon > 0$. Since $x_n$ goes to zero, at some point $n_0$ we have that $|x_n| < varepsilon$ if $n geq n_0$. Now, if $n geq n_0$, each $x_n$ is either zero, lies on some interval $left[frac{1}{M_n+1},frac{1}{M_n}right]$ with $M_n > 1/varepsilon$, and so by the definition of each $f_N$ then, we have that $|f(x_n)| = f(x_n) leq 1/M_n < varepsilon$. In any case, we have that $|f(x_n)| < varepsilon$ if $n geq n_0$, which proves that $f$ is continuous at zero.



        So far we have only defined $f$. We still ought to show that it is not of bounded variation, despite being continuous. Intuitively this is because the spikes add each a variation of no less that $1/N$ and the harmonic series diverges. Concretely, if $f$ were of bounded variation, we would have that for each $N geq 1$,



        $$
        V_0^1f = V_0^{1/(N+1)}f + sum_{i=1}^NV_{1/(i+1)}^{1/i}f = dots \
        dots = V_0^{1/(N+1)}f + sum_{i=1}^NV_{1/(i+1)}^{1/i}f_i geq V_0^{1/(N+1)}f + sum_{i=1}^Nfrac{1}{i} geq sum_{i=1}^Nfrac{1}{i}.
        $$



        Taking limit when $i to infty$ we have $V_0^1f = +infty$. Moreover, $f$ is not of bounded variation in any interval of the form $[0,c]$: if it were so, by refining partitions and taking limits we would have that



        $$
        V_0^1f = V_0^cf + V_c^1f,
        $$



        and since $f|_{[c,1]}$ is of bounded variation (we are interpolating linearly a finite amount of times), then $V_c^1 < + infty$ which would imply that $V_0^1f < + infty$, a contradiction.






        share|cite|improve this answer














        Hint: it is well known that functions of bounded variation are of the form $f = g-h$ with $g,h$ monotone. In particular, $f$ ought to be differentiable almost everywhere on $[0,1]$. However, functions such as the Weierstraß function (restricted to $[0,1]$) are continuous but differentiable nowhere. In particular it is not differentiable in any proper subinterval and so by the former, it is not of bounded variation there either.



        Edit: the original question was modified, and this does not answer the content of the post: what follows is a construction of a continuous function $f : [0,1] to mathbb{R}$ which is not of bounded variation in any subinterval $[0,c]$ with $c leq 1$, but for which $V_c^1f < +infty$.



        First, let's construct a function $[0,1] to mathbb{R}$ which is not of bounded variation.



        Consider $f_N : left[frac{1}{N+1},frac{1}{N}right]to mathbb{R}$ with interpolates linearly the points:
        $$
        (1/(N+1),0),left(frac{1/(N+1)+1/N}{2},1/Nright) text{ and } (1/N,0)
        $$



        A drawing should illustrate the idea: we are making spikes whose height decreases with $N$, and so does the length of the considered interval. At the endpoint of the interval $f_N$ is zero, and in the middle point it attains its maximum value which is $1/N$. Moreover, $f_N$ is piecewise monotone. Now the functions $(f_N)_{Ngeq 1}$ coincide at ${1/N : N in mathbb{N}}$ and so the function



        $$
        f(x) = f_N(x) text{, if } x in left[frac{1}{N+1},frac{1}{N}right]
        $$



        is continuous on $(0,1]$. Technicalities aside, we are just gluing the spikes together and justifying that the resulting function is indeed contiunous. Finally, we would like to extend $f$ to $[0,1]$. A necessary condition is for $f(0)$ to be zero, since we already have $f(1/N) = 0$ and $1/N to 0$. So let's check that



        $$
        f(x) = cases{f(x) x neq 0 \ 0 text{ otherwise}}
        $$



        is continuous on $[0,1]$. Take $x_n to 0$ and $varepsilon > 0$. Since $x_n$ goes to zero, at some point $n_0$ we have that $|x_n| < varepsilon$ if $n geq n_0$. Now, if $n geq n_0$, each $x_n$ is either zero, lies on some interval $left[frac{1}{M_n+1},frac{1}{M_n}right]$ with $M_n > 1/varepsilon$, and so by the definition of each $f_N$ then, we have that $|f(x_n)| = f(x_n) leq 1/M_n < varepsilon$. In any case, we have that $|f(x_n)| < varepsilon$ if $n geq n_0$, which proves that $f$ is continuous at zero.



        So far we have only defined $f$. We still ought to show that it is not of bounded variation, despite being continuous. Intuitively this is because the spikes add each a variation of no less that $1/N$ and the harmonic series diverges. Concretely, if $f$ were of bounded variation, we would have that for each $N geq 1$,



        $$
        V_0^1f = V_0^{1/(N+1)}f + sum_{i=1}^NV_{1/(i+1)}^{1/i}f = dots \
        dots = V_0^{1/(N+1)}f + sum_{i=1}^NV_{1/(i+1)}^{1/i}f_i geq V_0^{1/(N+1)}f + sum_{i=1}^Nfrac{1}{i} geq sum_{i=1}^Nfrac{1}{i}.
        $$



        Taking limit when $i to infty$ we have $V_0^1f = +infty$. Moreover, $f$ is not of bounded variation in any interval of the form $[0,c]$: if it were so, by refining partitions and taking limits we would have that



        $$
        V_0^1f = V_0^cf + V_c^1f,
        $$



        and since $f|_{[c,1]}$ is of bounded variation (we are interpolating linearly a finite amount of times), then $V_c^1 < + infty$ which would imply that $V_0^1f < + infty$, a contradiction.







        share|cite|improve this answer














        share|cite|improve this answer



        share|cite|improve this answer








        edited Nov 25 at 23:19

























        answered Nov 25 at 23:05









        Guido A.

        6,9561730




        6,9561730






























            draft saved

            draft discarded




















































            Thanks for contributing an answer to Mathematics Stack Exchange!


            • Please be sure to answer the question. Provide details and share your research!

            But avoid



            • Asking for help, clarification, or responding to other answers.

            • Making statements based on opinion; back them up with references or personal experience.


            Use MathJax to format equations. MathJax reference.


            To learn more, see our tips on writing great answers.





            Some of your past answers have not been well-received, and you're in danger of being blocked from answering.


            Please pay close attention to the following guidance:


            • Please be sure to answer the question. Provide details and share your research!

            But avoid



            • Asking for help, clarification, or responding to other answers.

            • Making statements based on opinion; back them up with references or personal experience.


            To learn more, see our tips on writing great answers.




            draft saved


            draft discarded














            StackExchange.ready(
            function () {
            StackExchange.openid.initPostLogin('.new-post-login', 'https%3a%2f%2fmath.stackexchange.com%2fquestions%2f3013501%2fa-continuous-function-on-0-1-that-is-not-of-a-bounded-veriation%23new-answer', 'question_page');
            }
            );

            Post as a guest















            Required, but never shown





















































            Required, but never shown














            Required, but never shown












            Required, but never shown







            Required, but never shown

































            Required, but never shown














            Required, but never shown












            Required, but never shown







            Required, but never shown







            Popular posts from this blog

            To store a contact into the json file from server.js file using a class in NodeJS

            Redirect URL with Chrome Remote Debugging Android Devices

            Dieringhausen